PAST PAPERS - DMH Flashcards

1
Q

Cognitive behavioral therapy is an important part of the therapeutic management of the
condition. Define what is meant by CBT and discuss its major components, and then how
we would use it in the holistic therapeutic management of patients with OCD. (15)

A

COGNITIVE BEHAVIORAL THERAPY

Cognitive - our thoughts, beliefs and attitudes

Behavioral - what we do, how we act

Main idea is that our thoughts have an effect on out behaviors and conversely our behaviors on our thoughts, and they can interact to create cycles and patterns in our lives. These cycles can be positive and productive, or negative and make dealing with changes or life stressors more challenging.

The theory is that making a change in what we are thinking or doing can have a lasting impact on how we are feeling and change these cycles.

The goal of CBT is helping the patient develop and awareness of what they are thinking and how these thoughts influence their behaviors.
A specific area of focus in anxious thoughts and assumptions is encouraging the patient to ask themselves whether the thoughts they have are accurate to reality or not. Ask the patient how those thoughts influence their behaviors?

Identify the thoughts, evaluate the thoughts (is this accurate, what is the evidence) and then look at different strategies for behavior in response to these thoughts.

Why will one person have different thoughts and behaviors to another?

  1. They have different core beliefs - therefore the strategy is changing those core beliefs will ultimately end up changing your mood.

Negative core beliefs are grouped into three categories:
- Helpless Core Beliefs
- Unlovable Core Beliefs
- Worthless Core Beliefs
- About the World
- About other people

  1. Rules and Assumptions (also known as intermediate beliefs)

(Think about “I should…” “I must” and “If….., then …”)

If struggling to determine the core belief, can work backwards from the negative thought.
When patients have negative thoughts, ask the question “If this is true, what does this say or mean about me?” Continue asking this question until you end up with the core belief.

How well did you know this?
1
Not at all
2
3
4
5
Perfectly
2
Q

Mr Kay is a 75-year-old retired teacher who is a widow, and lives on his own. He has been
living with hypertension and diabetes mellitus (for the last 20-years) which are well controlled.
His daughter approaches you because she wants to place her father in an old age home, and
sell the house he currently lives in. She brings him to you for a medical and mental assessment
as required by the old age home.

a) In a clinical setting how would you evaluate Mr Kay’s competency to engage in selling his
property? (15)

b) After your assessment you come to the conclusion that Mr Kay has mild cognitive deficits.
During your assessment Mr Kay informs you that he wants to make amendments to his
existing will. How would you evaluate his testamentary capacity? (10)

A

(A)
COMPETENCE - ability to make autonomous, informed decisions that are consistent with the person’s own lifestyle and attitude and take the necessary action to put these decisions into effect.

A person is competent if they are able to:
1. understand the material information
2. make a judgement about the information in light of their values
3. Intend a certain outcome
4. Communicate freely their wishes to caregivers and investigators

There is lack of contractual capacity (which would be needed to sell a property) if at the time of the contract, the person has a mental illness or intellectual disability, and this illness or disability affected his/her understanding such that he/she did not know the nature and consequences of the transaction. It is task dependent - can have capacity for some tasks and not others.

COMPETENCE ASSESSMENT MNEMONIC:
SOCCOUR
S - Situation (does the person know, understand and remember the situation)
O - Options (is the person aware of their options)
C - Consequences (is the person aware of the consequences of their decision)
C - Consistency (does the person show consistency in their decisions, understanding and values)
O - Opinion (to get the opinion of those that know the person, collateral information)
U - Undue influence (is there reason to suspect that this person is under the influence of others)
R - Reasons (does the person give reasons for their decision)

(B)
DEFINITION OF CAPACITY deals with two key elements:
1. a person’s ability to assimilate relevant facts
2. appreciation or understanding of his/her situation as it relates to the facts

  • Evaluation of testamentary capacity commonly involves a variety of mental health professionals including psychiatrist, psychologist, social worker, occupational therapist.
  • collateral information needed - ADLS
  • diagnosis and differential diagnoses
  • evaluation of the severity of the condition
  • determine the ability to perform the task
  • having testamentary capacity requires greater mental capacity than for other contracts as by law, a person much have a “sound and disposing mind and memory”
  • all adults are presumed to have testamentary capacity - those who challenge it need to demonstrate that the testator did not know the consequences of their conduct when executing a will.

Necessary Functions to Have testamentary capacity:
- understand nature of act
- understand and recollect nature, extent and situation of property
- remember and understand relations and those affected by the will
- understand the dispositions they are making
- know how elements above relate to form an orderly plan of property distribution
- no active symptoms of major mental illness
- no undue influence from other parties.

WHAT DOES THE DOCTOR ACTUALLY DO?
Before Death
- Full history and MSE
- Cognitive testing (MOCA, MMSE)
- Hospital records and previous investigations
- Collateral information regarding the facts about the estate
- Include in the report the testator’s awareness of the estate, current will and reasons for wanting to change it.

After Death
- hospital records and previous investigations
- full history and an MSE around the time of the will change if available
- videos of proof at the time of changing the will
- affidavit about the will.

How well did you know this?
1
Not at all
2
3
4
5
Perfectly
3
Q

How does one make an application for curatorship and what are the different types of curators the court may appoint?

A
  • application is made to the high court to appoint a curator
  • applicant is generally a family member but can be any interested party
  • the application must be accompanied by affidavits from two medical practitioners - one a psychiatrist

The court is requested to address three aspects in its enquiry
1. to declare the person of unsound mind and incapable of managing his/her affairs

  1. to appoint curator ad litem (act on behalf, especially during legal proceedings - can also be appointed for minor)
  2. to appoint a curator bonis (financial affairs) or curator personae or both

A curator bonis may perform the following functions amongst others (usually a lawyer):

  • Manage the maintenance or support of the relevant person;
  • Purchase or acquire immovable property on behalf of the relevant person;
  • Manage the relevant person’s business affairs; and
  • Administer the relevant person’s estate or assets.

Curator personae - appointed to oversee the personal and wellness affairs of an incapacitated person, welfare, decision making regarding healthcare, accommodation and general well being.

How well did you know this?
1
Not at all
2
3
4
5
Perfectly
4
Q

a) Describe the characteristics of a panic attack. (6)

b) List five differential diagnoses for a panic attack. (5)

c) Differentiate a panic attack from a panic disorder. (4)

d) Discuss the treatment of a panic disorder. (10)

A

(a) A abrupt surge of intense fear or discomfort that reaches maximal intensity after 1-2 minutes and usually lasts for 10-20 minutes.
Surge can occur from a state of calm or from a state of anxiety.
A panic attack has both physical and psychological symptoms.
Must have 4 of the following symptoms: (STUDENTS FEAR 3Cs)
S - sweating
T - trembling/shaking
U - unsteadiness/dizziness
D - derealization/depersonalization
E -elevated heart rate, palpitations, pounding heart
N - nausea and abdominal discomfort
T - tingling sensation and numbness
S - shortness of breath and smothering
Fear of losing control, going crazy or dying.
3Cs - chest pains, choking, chills

Don’t include culture-specific symptoms: tinnitus, neck soreness, headache, uncontrollable screaming or crying.

(b) Heart attack, gastritis, asthma attack, COPD exacerbation, stimulant intoxication, hyperthyroidism.

(c)
- Panic disorder is a lot less prevalent than the occurrence of a panic attack
- most common in females
- generally, follows a chronic relapsing remitting course

CRITERION A - characterized by recurrent unexpected panic attacks
CRITERION B - at least one of the attacks if followed by at least one month of
1. persistent concern or worry about additional panic attacks or their consequences
AND/OR/
2. significant maladaptive change in behavior related to these attacks - avoidance type.

CRITERION C - disturbance not attributable to physiological effects of a substance or medical condition

CRITERION D - disturbance not better explained by another mental disorder

(d)
PHARMA - SSRIs - inhibit amygdala hyperactivity thus modulating the conditioned fear network. Also decrease noradrenergic action which relieves some symptoms such as palpitations.
Second line - different SSRI or SNRI, TCAs, adding a benzodiazepine in the short term for acute rescue of panic attacks when they occur.
Only switch to second line when the initial drug dosage has been optimized and adherence to treatment established, assessment of comorbid conditions (psych and med) performed.

PSYCHO - CBT is mainstay of treatment. Also available are relaxation therapies and desensitization therapy.

How well did you know this?
1
Not at all
2
3
4
5
Perfectly
5
Q

Despite years of warnings about the hazards of prescribing benzodiazepines, these drugs
continue to be used at a higher rate than what is considered appropriate. List some of
the recognized good practices you would advise to Primary Health Care doctors when
prescribing benzodiazepines. (8)

A
  • benzodiazepines should not be prescribed as anxiolytics or hypnotics for longer than 4 weeks to avoid dependence, tolerance and severe withdrawal symptoms
  • intermittent use (not every day) at the lowest possible is also recommended
  • should not be prescribed in the elderly population
  • careful tapering of those who are already on long term benzodiazepine is recommended. Taper over at least a 10-week period but possibly much longer, case dependent.
  • for those on shorter acting benzodiazepines, an approach to tapering and stopping could be to cross taper to diazepam which has a longer half-life and therefore associated with less severe withdrawal symptoms when eventually stopped.
  • patients should be carefully counselled on the risk of benzodiazepines - increased falls, dangerous to operate machinery, sedation, accidental overdose if used in excess, long term risk of cognitive impairment, delirium in elderly patients, withdrawal on stopping.
  • long acting benzos are always a better choice to short acting
  • recommends that benzos should not be used as mainstay of treatment in GAD, panic disorder or social phobia except for short term use in acute crisis states.
  • because of the addictive nature of benzos, there is a shift to the use of Z-drugs for the treatment of insomnia or alternative methods
How well did you know this?
1
Not at all
2
3
4
5
Perfectly
6
Q

What is the MOA of benzodiazepines?

A
  • Bind to GABA Type A which is a major receptor implicated in inhibition in the CNS and sedation
    • These receptors are ligand-gated chloride ion channels
    • When GABA binds to the receptors, it increases the amount of chloride current generated by the receptor
    • Benzos increase the frequency of the channel opening which augments the inhibitory effect of the GABA

Metabolized by CYP3A4

How well did you know this?
1
Not at all
2
3
4
5
Perfectly
7
Q

What are some factors associated with more severe withdrawal symptoms in those treated with benzodiazepines?

A
  • Brain damage
  • Alcohol addiction
  • EEG abnormalities
  • abrupt discontinuation after regular use
  • longer duration of use prior to discontinuation
  • higher doses
  • shorter half-life
How well did you know this?
1
Not at all
2
3
4
5
Perfectly
8
Q

Who is the curator ad litem for state patients?

A

The Department of Public Prosecutions

How well did you know this?
1
Not at all
2
3
4
5
Perfectly
9
Q

What is involved in a curatorship assessment?

A

Full history, MSE and applicable investigations to provide a diagnosis.

Investigations include MMSE and neuro-psychological evaluation if required.

An OT may be required to assess occupational aspects of the persons functioning.

If collateral information is deemed insufficient, or if family discontent threatens to hamper
proceedings, social worker intervention is recommended.

Important to establish the persons general level of functioning.

In addition, assess for financial competence:
◦ Knowledge of income.
◦ Knowledge of expenses.
◦ Ability to handle everyday financial transactions (serial 7’s, balance cheques, counting change,
paying bills).
◦ Ability to delegate financial wishes.

How well did you know this?
1
Not at all
2
3
4
5
Perfectly
10
Q

Compare and contrast impairment vs disability

A

Impairment is the alteration of normal functional capacity due to a disease, and is assessed
by medical means after a diagnosis has been established, and appropriate and optimal
treatment applied.

In practical terms, impairment assessment entails examining the diagnosis and current and
future treatment options before determining on medical grounds which functions the
person is still able to do and which not.

Occupational therapists play an important role prior to taking a final decision regarding
extent of impairment.

Disability is the alteration of capability to meet the personal, social or occupational
demands due to an impairment, and is assessed by non-medical means.

Assessing disability entails assessing the extent of a person’s impairment in conjunction
with their job description, policy disability clause condition and personal factors such as
education, experience etc.
Disability assessment is a legal and not a medical decision, taken by a panel of experts
including a:

◦ medical advisor,
◦ legal advisor,
◦ claims consultant.

The psychiatrist treating the patient does not usually possess all the information, and is
therefore in no position to express an opinion on disability.
The psychiatrist’s function is to assess the areas of impairment and to indicate whether it is
permanent or not, while the actual decision regarding disability is taken by a separate panel of assessors

How well did you know this?
1
Not at all
2
3
4
5
Perfectly
11
Q

Miss X, a 48-year-old female presents to the emergency department with a 1-week history of
acute confusion, disorientation, and disruptive and disorganized behavior. She is known to
be HIV positive, with a CD4 count of 271. She was started on antiretroviral treatment two
months ago. She has no previous psychiatric history. There is no history of substance use,
and no family history of a mental illness.

a) List the differential diagnoses for her acute symptoms and motivate (factor for and
against) each diagnosis. (8)

Her family reports that she has gradually become slower and clumsier in the last 2-years and
is starting to become forgetful. She was fired from her clerical job last year for poor work
performance.

b) List the four most likely differential diagnoses for her more long-standing neuropsychiatric
symptoms. (4)

c) Discuss the use of antipsychotics in patients with HIV. (13)

A

a)
In this case:
- CNS infection - TBM, CCM - possibly with immune reconstitution syndrome
- Space occupying lesion - possible tuberculoma
- Side effect of HAART therapy
- Organic psychiatric disease
- HIV effects on the CNS - HIV dementia

Factors Causing Psychiatric Symptoms in PLWH (people living with HIV)

Primary psychiatric disorders
Neurobiological changes caused by HIV in the CNS
Other infections or CNS tumours
Antiretroviral drugs and other medical treatments
Alcohol or substance misuse
Adverse psychosocial factors (stigma)

b)
Mild Neurocognitive Disorder Due to HIV infection

Major depressive disorder

Major neurocognitive disorder due to HIV infection

Other CNS infection or space occupying lesion??

c)
General Prescribing Principles
- start with low dose and titrate to tolerability and effect
- select simplest dosing regimen possible
- select agent with fewest side effects
- MDT preferably
- those infected with HIV are more susceptible to EPSEs due to HIV invasion of the basal ganglia - therefore SGAs such as quetiapine, risperidone and aripiprazole have been suggested as first line in psychosis not due to dementia and delirium.
- Monitoring of metabolic side effects and QT prolongation may be more important in those on APs and HAART
- those PLWH who have psychiatric disorders such as NCDs, SCZ or MDD may have worse compliance to both HAART and psychotropic medications. Therefore, greater need to support compliance.
- Escitalopram and Citalopram have lower risk of pharmacokinetic interactions in those on HAART (with ECG monitoring)
- Mirtazapine may be good choice in those patients who are wasted and also those with comorbid methamphetamine use
- TCAs and MAOIs are not recommended
- PLWH are more sensitive to side effects of mood stabilizers such as neurotoxicity and lithium, especially if they have neurocognitive dysfunction. Be aware of possible renal impairment due to HIV. Lithium should be avoided in advanced HIV disease.
- Carbamazepine should be avoided as drug-drug interactions are significant.
- Valproate can be an alternative but close monitoring required due to risk of hepatotoxicty and other drug drug interaction. Best avoided in those on other hepatotoxic drugs - nevirapine, rifampicin. Mood stabilizing antipsychotics are good options in mood disorders.

  • Secondary Mania (“HIV mania”) - reports of secondary mania typically occurring in advanced illness such as HIV-associated neurocognitive disorders or CNS opportunistic infections. May respond to SGAs.
How well did you know this?
1
Not at all
2
3
4
5
Perfectly
12
Q

Polypharmacy has serious potential risks and increases the side effect burden both
acutely and over the long term. Discuss strategies you would employ to limit
polypharmacy and over-prescribing during the maintenance phase of treating a severe
mental illness. (9)

A
How well did you know this?
1
Not at all
2
3
4
5
Perfectly
13
Q

Long-acting Injectable Antipsychotics (LAI) were developed in response to high rates of
poor adherence to oral formulations. Briefly discuss.

i) Advantages of LAIs. (4)

ii) Limitations of LAIs. (4)

A
How well did you know this?
1
Not at all
2
3
4
5
Perfectly
14
Q

A 20-year-old female comes to see you after failing her final examinations. She reports feeling
sad and lonely, after recently breaking up with her boyfriend. During the interview you find her
to be very tearful. You make a diagnosis of Major Depressive Disorder (MDD).

a) Tabulate the clinical features associated with Major Depressive Disorder under the two
different categories i.e. Neurovegetative and Other. (8)

b) Give the 1st line pharmacotherapy you would prescribe, including dose ranges, duration
of treatment and the three most common side-effects associated with this drug. (6)

c) Discuss how you would differentiate between MDD and grief. (11)

A
How well did you know this?
1
Not at all
2
3
4
5
Perfectly
15
Q

A 9-year-old boy who has been continent of urine since the age of 3-years has now started
wetting the bed. His father has lost his job 2-months ago and he has a 15-year-old sister who
has been caught smoking cannabis at school. You are the GP for this family and the boy has
been brought to your practice as his parents are very frustrated by his bedwetting.

a) Discuss how you would do an assessment on any child presenting with enuresis. (10)

b) Discuss the management of the child in the case scenario above. (15)

A
How well did you know this?
1
Not at all
2
3
4
5
Perfectly
16
Q

You are required to assess a 24-year-old woman who was admitted overnight after taking an
overdose of “sleeping pills”, which she had taken subsequent to having “a few drinks”, after
being told that her boyfriend was seen with another woman. You notice a number of scars on
her wrists.

a) Describe what factors you would consider in doing a suicide risk assessment. (10)

b) How would you assess whether she has an alcohol use disorder? (5)

c) List 5 features of borderline personality disorder. (5)

d) She denies that the scars on her wrists were due to suicide attempts, but says that she
sometimes feels an urge to hurt herself. Briefly discuss the significance of this with
regards to borderline personality disorder. (5)

A
How well did you know this?
1
Not at all
2
3
4
5
Perfectly
17
Q

Mr A, a 24-year-old patient known with schizophrenia comes to see you in the casualty
department. He reports that he was charged with theft after stealing a bicycle and is worried
that he will be sentenced to jail. He says he stole the bicycle as the voices told him to do so.
He requests admission to hospital (it is clear he hopes this will prevent him from going to jail).
According to his previous notes he always has residual psychotic symptoms on follow up,
despite an adequate dose of Clozapine.

a) Discuss which options are available for admission under the Mental Health Care Act
(MHCA) and the factors that should be considered when doing so. Motivate the form of
admission required for this patient. (10)

b) Discuss the ethical principles, as they apply to this case, under the following headings
i) Beneficence.
ii) Non-maleficence.
iii) Autonomy.
iv) Justice. (15)

A
How well did you know this?
1
Not at all
2
3
4
5
Perfectly
18
Q

A 50-year-old recently divorced man presents with major depression. This is his first
presentation to psychiatry. There is a family history of two uncles having committed suicide.
His marriage disintegrated after the death of his son in a motorcycle accident. He bought his
son the motor cycle for his eighteenth birthday present. His son died a week after his birthday.

He lives alone and has a very stressful job as an auditor.

a) How would you assess for suicidal risk in this patient? (10)

b) What would be your bio-psycho-social clinical management of this patient? (15)

A
How well did you know this?
1
Not at all
2
3
4
5
Perfectly
19
Q

With regard to cannabis use and psychotic disorders:

a) Explain the biological mechanisms underlying the association between cannabis and
psychosis. (5)

b) Specify risk factors that may increase the likelihood of developing schizophrenia after
cannabis use. (5)

c) Describe the clinical course (acute and long-term) of cannabis use. Include a description
of the clinical symptoms of intoxication and withdrawal in the answer. (10)

d) Discuss the role of cannabis as a potential cause of schizophrenia. (5)

A
How well did you know this?
1
Not at all
2
3
4
5
Perfectly
20
Q

a) A 30-year-old lady presents to the emergency department with a manic episode. She is
aggressive and she wants to jump from the hospital roof as she believes that she can fly.
Her husband tells you that she is known with bipolar I disorder and that she was doing
very well until she stopped her treatment four months ago when they decided to start a
family. She is now 6 weeks pregnant. He demands that you put her back on the
medication she was on before: Sodium Valproate 500mg twice daily and Olanzapine
10mg daily. Describe your approach to the assessment and acute management of this
patient. (15)

b) Write short notes on the use of the following medication in pregnancy:
i) Lithium. (5)
ii) Sodium Valproate. (5)

A
How well did you know this?
1
Not at all
2
3
4
5
Perfectly
21
Q

A 55-year-old male is accompanied by his family to the emergency department with a history
of sudden onset of confusion and inappropriate behaviour. He is extremely disruptive and
agitated. It is difficult to contain him to be evaluated. He is wandering around the room, shouting
and insulting patients waiting to be seen for no apparent reason. Attempts to calm him down
are met with physical aggression especially toward furniture in the cubicle. He is suspicious of
everybody around him and appears to be responding to visual hallucinations. It is difficult to
obtain a coherent history as well as conduct a mental state examination due to impairment in
his attention and distractibility. He also appears to be disoriented to time and place. Your
colleague advises you to admit him to the psychiatric ward for treatment of his psychosis. Upon
reassessing the patient, he is found to be asleep and you have difficulty rousing him even
though he did not receive any sedation in the interim and there was no history of substance
use prior to admission to the hospital.

a) The information in the case is suspicious of which DSM 5 diagnosis? (2)

b) What are the DSM 5 criteria of the above condition? (10)

c) Do you agree with the colleague’s advice to admit the patient to the psychiatric ward for
treatment of his psychosis? Substantiate your answer. (3)

d) You realise that appropriate management of the condition requires further investigation
into the underlying cause. Collateral information and a thorough physical examination
yield no definitive answer. What special investigations would you consider essential for
the work-up of the patient and substantiate the reason for the investigations? (10)

A
How well did you know this?
1
Not at all
2
3
4
5
Perfectly
22
Q

An 11-year-old child presents with a history of disruptive and aggressive behaviour; and poor
school performance.

a) What information would you seek on history? (7)

b) Discuss your differential diagnosis. (8)

c) Outline your management of this patient if you make a diagnosis of Attention Deficit
Hyperactivity Disorder. (10)

A
How well did you know this?
1
Not at all
2
3
4
5
Perfectly
23
Q

A 77-year-old male is brought to you by his family. He has no previous psychiatric history; and
besides hypertension and type II diabetes mellitus, he has no other significant medical history.
After assessing him, you suspect he may have a neurocognitive disorder due to vascular
disease.

a) Tabulate the differences between mild and major neurocognitive disorder. (5)

b) List the cognitive domains that may be affected, and for each, describe the possible
complaints the family may report on history that would indicate impairment in that domain.
(9)

c) Discuss the role of pharmacotherapy in the management of this patient. (11)

A
How well did you know this?
1
Not at all
2
3
4
5
Perfectly
24
Q

Write short notes on the following:
a) Mental health services are delivered at 3 levels namely primary, secondary and tertiary
levels. What are the components of primary health care and what are the staff
complements of the components? (15)

b) Confidentiality refers to the medical premise that binds the medical practitioners to keep
secret all information divulged by patients during a doctor-patient interaction. When are
you allowed to breach doctor-patient confidentiality in a mental health setting? (5)

c) What is your involvement as a general practitioner in the clinical management of persons
referred by the Criminal Justice system? (5)

A
How well did you know this?
1
Not at all
2
3
4
5
Perfectly
25
Q

Mr. Smith is a 26-year-old male with a 5 year history of Schizophrenia. He is brought to the
emergency department of your district hospital by his parents against his wishes. He has not
taken his treatment for several months and describes a 6 weeks history of auditory
hallucinations telling him that he is not safe, as well as persecutory delusions that he is in
danger from the South African police services (SAPS) as they are involved in a conspiracy with
Interpol to arrest him for crimes he has not committed. He refuses hospital admission as he
does not believe that he is unwell. He lost his job a few weeks ago due to his “odd behaviour”
at work.

a) Outline the factors which may influence his non-adherence to medication. (10)

b) How may stigma impact the lives of people living with mental illness? (8)

c) Following his admission as an involuntary user under the Mental Health Care Act, Mr.
Smith requires a dental extraction and significant dental repair due to a dental abscess.
His dentist recommends that the procedure is conducted under general anaesthesia due
to the extent of the work and asks you to sign Mr. Smith’s consent form as he is an
involuntary user. This is not an emergency procedure. Discuss your approach to this
request. (7)
[25]

A
How well did you know this?
1
Not at all
2
3
4
5
Perfectly
26
Q

Write notes on the important differences between Schizophrenia and
a) Major Depression with psychotic features. (9)
b) Delirium. (7)
c) Alcohol withdrawal with perceptual disturbances. (9)

A
How well did you know this?
1
Not at all
2
3
4
5
Perfectly
27
Q

Substance use disorders are common in South Africa and Motivational Interviewing and Brief
interventions play a vital role in the therapeutic process of managing these conditions
a) Define motivational interviewing and discuss the key principles of the stages of change
model. (15)
b) What is your understanding of the brief intervention model called FRAMES? (10)

A
How well did you know this?
1
Not at all
2
3
4
5
Perfectly
28
Q

a) Describe the DSMV diagnostic criteria for General Personality Disorder i.e. the general
definition which applies to each of the 10 specific personality disorders described in
DSMV. (6)

b) Describe 6 diagnostic criteria features of Borderline Personality Disorder as described by
DSMV. (6)

c) Write short notes on important principles which apply to the use of medication in the
treatment of patients with Borderline Personality Disorder. (4)

d) Write short notes on the differences between Obsessive Compulsive Disorder and
Obsessive Compulsive Personality Disorder. (4)

e) Write short notes on the differences between Borderline Personality Disorder and Bipolar
Mood Disorder. (5)

A
How well did you know this?
1
Not at all
2
3
4
5
Perfectly
29
Q

In the clinical management of Obsessive Compulsive Disorder
a) Discuss an approach to the pharmacological management of the condition. (10)

A
How well did you know this?
1
Not at all
2
3
4
5
Perfectly
30
Q

A 9-year-old boy has started wetting the bed after being continent since the age of 3 years. He
stole R50 from the teacher’s purse and has been bullying smaller children at his school. He
uses foul language when confronted by his teachers and ran away from home and slept in the
local park for one night. His school is threatening to expel him due to pressure from the parents
of the children he has bullied
a) Discuss your approach to assessing and managing this patient and include a differential
diagnosis. (15)
b) Discuss how you would manage Enuresis in a patient. (10)

A
How well did you know this?
1
Not at all
2
3
4
5
Perfectly
31
Q

You are a medical officer working in an acute setting. A 40-year-old woman has been admitted
after her third suicide attempt, all of which have been by taking an overdose of tablets
a) How would you evaluate her to decide whether she is serious about her intention to kill
herself? (15)
b) You decide that she does pose a major suicidal risk but she is adamant that she will not
remain in hospital. How will you proceed to ensure her stay in hospital even though it will
be against her will? (10)

A
How well did you know this?
1
Not at all
2
3
4
5
Perfectly
32
Q

You are working as a locum for a GP when a 33-year-old female client comes to see you.
She has numerous physical complaints ranging from headaches, earache, pain on
swallowing, sore throat, dysuria, chest pain radiating to the abdomen and insomnia for a
month. She asks for ‘the strongest pain killers and sleeping tablets’. She has a thick clinical
file and the last 4 consultations have revealed no major physical illnesses. A full examination
reveals that her vital signs are all within normal range and except for slight oropharangeal
inflammation, she appears otherwise physically healthy
a) What is the possible differential psychiatric diagnosis? (2)
b) What are common causes of insomnia? (5)
c) Discuss the non-pharmacological management of chronic insomnia. (10)
d) Discuss why you would or would not give analgesics to this patient. (8)

A
How well did you know this?
1
Not at all
2
3
4
5
Perfectly
33
Q

a) Specifiers for Major Depressive Disorder give a clear indication of severity and
longitudinal aspects of the disease process. Name the specifiers of Major Depressive
Disorder. (8)

b) Psycho-education involving patients and their families suffering from Major Depressive
Disorder are important components of the treatment process. What are the issues that
should be included in this process? (10)

c) What are the things you need to exclude clinically, when you conclude that an
antidepressant is ineffective in the treatment of a patient? (7)

A
How well did you know this?
1
Not at all
2
3
4
5
Perfectly
34
Q

You are responsible for the treatment of a male patient aged 20-years with a first presentation
of Schizophrenia on an outpatient basis
a) Name and compare the side effect profiles and indications for use of the typical (first
generation) versus the atypical (second generation) antipsychotic medications. (12)
b) After 4-6 weeks of antipsychotic medication, the patient remains floridly psychotic.
Discuss the possible reasons. (13)

A
How well did you know this?
1
Not at all
2
3
4
5
Perfectly
35
Q

A 33-year-old housewife presents with excessive worry about her finances, household and the
welfare of her three young children (aged 7 years, 4 years and 3-months-old). She often thinks
and worries whether she will live long enough to see them grow and be able to provide for
them since her husband died 7 months ago. She admits to drinking wine to help her cope. She
further reports poor sleep because her mind does not “switch off” at night and wakes up tired;
eats a lot of food but is unable to finish all her house chores as before; and struggles to get
her children ready for school in time
a) Discuss the differential diagnoses in this patient. (8)
b) After thorough examination and investigations you make a diagnosis of generalised
anxiety disorder (GAD). Discuss the core diagnostic features of generalised anxiety
disorder. (7)
c) Discuss the non-pharmacological management of GAD. (10)

A
How well did you know this?
1
Not at all
2
3
4
5
Perfectly
36
Q

a) Discuss the clinical features of Borderline Personality Disorder. (12)
b) In clinical practice, it may be difficult to differentiate between Borderline Personality
Disorder and Bipolar Disorder, Type 2. Discuss some of the characteristics of these
two disorders that would assist clinicians in differentiating between them. (7)
c) Discuss the role of psychotropic medications in the management of Borderline
Personality Disorder. (6)

A
How well did you know this?
1
Not at all
2
3
4
5
Perfectly
37
Q

A 22-year-old actuarial student presents to you with a history of poor attention and
forgetfulness. He did very well for the first three years of his studies, but he has failed several
modules during the last six months and one of his lecturers advised him to see a doctor. He
has been getting increasingly forgetful during the last year but he attributes these symptoms
to “stress”. He says that he is HIV positive and on antiretroviral treatment (ART) which he
receives from the local clinic
a) Discuss your approach in order to make a diagnosis in this patient. (13)

b) Briefly discuss the clinical presentation (including diagnostic criteria) of the HIV-
associated neurocognitive disorders (HAND). (5)

c) The patient admits to you that he has not been taking his ART as prescribed because
he often forgets to take his treatment. His CD4 is 205 and his viral load is high. Discuss
the further management of this patient. (7)

A
How well did you know this?
1
Not at all
2
3
4
5
Perfectly
38
Q

In our daily practice of psychiatry we are faced with an alarming rate of patients who are non-
compliant on medications

a) Briefly discuss the factors that influence compliance under the following headings
i) Patient related.
ii) Environmental related.
iii) Physician related.
iv) Treatment related. (10)
b) Briefly discuss how you would evaluate /measure compliance in your patients. (5)
c) Discuss your approach to improving compliance. (5)

A
How well did you know this?
1
Not at all
2
3
4
5
Perfectly
39
Q

A 28-year-old woman consults you because she is concerned about her 8-year-old son. The
woman has been diagnosed with depression and she has heard that depression can run in
families. Her son has been tearful and withdrawn at home and he is refusing to finish his meals.
Write short notes on the following
a) List the possible differential diagnoses in the child for the symptoms described by this
mother. (6)
b) Risk factors associated with the development of depression in childhood. (6)
c) The management of childhood depression. (13)

A
How well did you know this?
1
Not at all
2
3
4
5
Perfectly
40
Q

A 45-year-old man presents to you with a 3 month history of a low mood, headache, anhedonia
and suicidal ideation. He reports that he has recently increased his alcohol consumption to
cope with the stress of his impending divorce
a) Discuss your differential diagnoses for this patient. (5)
b) Discuss what investigations you would conduct in this patient. (5)
c) Discuss the patient’s suicide risk. (5)
d) Briefly discuss your approach to the further management of this patient. (10)

A
How well did you know this?
1
Not at all
2
3
4
5
Perfectly
41
Q

You provide a service as a medical officer to a PHC clinic in a peri-urban township. The primary
care nurses ask you to assist them in managing agitated and violent patients, particularly
adolescents and young adults. You have agreed to conduct a workshop on the overall
management of aggression and to draw up a guideline for emergency management by the
nursing staff. With your audience in mind, briefly discuss the following
a) Causes of aggression (other than criminal behaviour). (8)
b) Recognition of imminent violence and verbal de-escalation. (8)
c) The requirements, in terms of the Mental Health Care Act, for the South African Police
Services (SAPS), clinic staff and the Mental Health Review Boards with regards to
aggressive behaviour. (9)

A
How well did you know this?
1
Not at all
2
3
4
5
Perfectly
42
Q

a) Compare and contrast the clinical features of an Adjustment Disorder (with
depressed mood) and a Major Depressive Episode. (50)
b) Discuss the management of Adjustment Disorder with depressed mood. (50)

A
How well did you know this?
1
Not at all
2
3
4
5
Perfectly
43
Q

Write short notes on the clinical presentation and management of the following medication
emergencies
a) Neuroleptic induced acute dystonia. (30)
b) Serotonin syndrome. (30)
c) Neuroleptic malignant syndrome. (40)

A
How well did you know this?
1
Not at all
2
3
4
5
Perfectly
44
Q

28-Year-old Ms Williams presents to your general practice requesting diazepam for her
longstanding sleep problems. She tells you not to worry, she is educated about the side
effects as she has been taking them for a long time, but needs them now as her insomnia
has worsened. She is in a crisis because she fears her current relationship of three weeks
is threatened, possibly due to her anger outbursts; she is desperate to hold on. She is tired
of her multiple intense short-term relationships that start off so wonderfully and end so
horribly. She often feels very sure of herself, but these feelings do not last as she then feels
anxious, down and doubts herself. She has much difficulty controlling her feelings of anger
and emptiness for as long as she can remember, especially when having to deal with her
dad who was emotionally and physically abusive to the family, before he left them. You
notice that she has scars across her forearm in various stages of healing. On questioning,
you find that she had a recent parasuicide attempt by overdose due to relationship
problems and financial crises arising from overspending. Mental state examination reveals
that she is euthymic and apsychotic
a) What is Ms Williams’ most likely psychiatric diagnosis and what criteria does she
present with to substantiate this? (20)
b) This disorder has comorbid diagnoses that seem to be apparent here. What are
they? (10)
c) What is your approach to the biopsychosocial management of this case, highlighting
needs to be addressed? (30)
d) What does dialectical behaviour therapy encompass – what are the goals of
therapy? (10)
e) How would you manage Ms Williams’ long-term sleep problem? (30)

A
How well did you know this?
1
Not at all
2
3
4
5
Perfectly
45
Q

A 40-year-old male is brought to the casualty department by the police, with a history of
agitation and confusion. The police report that he was arrested 3 days ago for drunken
driving and has been in police custody since then. The police are able to give you his family
contact details and you discover that he is a heavy drinker. He has no significant medical
history and no evidence of injuries
a) Name and define the most likely diagnosis, and describe the clinical features
associated with this diagnosis. (15)
b) Explain in detail your holistic management of this patient as follows
i) Acute management. (40)
ii) Long-term management. (20)
c) Write short notes on Wernicke’s encephalopathy. (25)

A
How well did you know this?
1
Not at all
2
3
4
5
Perfectly
46
Q

You are working as a medical officer in a primary health care clinic. For many decades your
district has always had a community psychiatry clinic, situated and functioning separately
from your primary health care clinic. Mental health care conditions have always been
managed in the community psychiatry clinic and general health conditions have always
been a responsibility of the primary health clinic. Your district manager intends integrating
the community psychiatry clinic into the primary health clinic. Write short notes on the
following
a) The concerns your fellow medical officers, nurses, and mental health care users and
their families may have with this integration that is being planned. How can these
concerns be addressed? (50)
b) Possible advantages of this planned integration. (30)
c) Discuss stigma in relation to this planned integration and explore if it is likely to
increase or reduce stigmatisation of the mental health care users. (20)

A
How well did you know this?
1
Not at all
2
3
4
5
Perfectly
47
Q

A 7-year-old boy is brought by his mother to you at your hospital out-patient clinic. She
reports that he is wetting the bed every night
a) What is the normal sequence of developing control over bowel and bladder functions
during childhood? (15)
b) Describe the DSM 5 criteria that must be fulfilled to make a diagnosis of enuresis.
(25)
c) List five medical causes that may account for the clinical presentation. (10)
d) Discuss the appropriate investigations that one needs to conduct in order to exclude
medical causes. (10)
e) Discuss the holistic management of this child once all medical causes for the
bedwetting have been excluded. (40)

A
How well did you know this?
1
Not at all
2
3
4
5
Perfectly
48
Q

Ms M, a 25-year-old female, was arrested by the police for killing her one-year-old baby.
Before trial commencement, her attorney stated to the court that he could not get
instructions from her as she was acting strangely. Furthermore, her family had reported a
previous psychiatric history to the magistrate. The magistrate approaches you, a medical
officer at the local district hospital, to do a preliminary assessment on Ms M and make
recommendations to the court

4

a) Discuss how you would assess Ms M in order to provide the necessary
recommendations. (40)
b) Based on the available information, discuss your recommendations to the court and
give reasons. (30)
c) Describe what is required from the psychiatrist in terms of section 77 and section 78
of the Criminal Procedure Act No 51 of 1977. (30)

A
How well did you know this?
1
Not at all
2
3
4
5
Perfectly
49
Q

Write short notes on the following
a) Components of informed consent. (20)
b) Conditions under which doctor-patient confidentiality may be breached in the mental
health setting. (30)
c) Differential diagnosis in a 64-year-old female presenting with insomnia. (50)

A
How well did you know this?
1
Not at all
2
3
4
5
Perfectly
50
Q

An 18-year-old male presents with a first episode of psychosis. His brother was diagnosed
with schizophrenia two years ago. The parents would like to know if their 18-year-old has
schizophrenia as well
a) Write short notes on the questions that you would pose to the parents to elicit
prodromal symptoms suggestive of a diagnosis of schizophrenia. (10)
b) The patient is treated with Haloperidol. Five days after treatment initiation there is
concern of Neuroleptic malignant syndrome (NMS). Write short notes on how you
would make the diagnosis of NMS and your management thereof. (15)

A
How well did you know this?
1
Not at all
2
3
4
5
Perfectly
51
Q

A man brings his elderly mother to you with the complaint that she has been getting
increasingly forgetful over the past year. He is concerned that she might have Alzheimer’s
disease
a) Explain what your approach to this patient would be in order to make a diagnosis.
(15)
b) List five of the most likely and potentially treatable medical conditions that could
contribute to / cause the complaint of forgetfulness. (5)
You exclude medical conditions and make a diagnosis of Major Neurocognitive Disorder
according to the DSM 5
c) Alzheimer’s disease is one possible cause of a Major Neurocognitive Disorder. List
five other most common causes in a South African setting. (5)

A
How well did you know this?
1
Not at all
2
3
4
5
Perfectly
52
Q

A 28-year-old male is brought in by the police at 1am in the morning to the emergency unit,
where you are on call. They give a history that they were called by community members to
come and assist as he was roaming in the streets, shouting profanities and whenever
somebody approached him he would react violently, lashing out at them. They decided
to bring him to hospital as he appeared to be “confused”. The patient is now sitting in the
waiting area in hand-cuffs, mumbling away and would ever so often swear at people
staring at him. From the above clinical history
a) List the most likely differential diagnoses taking into consideration the history above.
(10)
b) Discuss the appropriate approach to the management of an aggressive and violent
patient. (15)

A
How well did you know this?
1
Not at all
2
3
4
5
Perfectly
53
Q

a) Define ‘insomnia’ and describe various types of insomnia. (5)

b) Write brief notes on the recommended general approach and non-
pharmacological management of insomnia. (10)

c) Write brief notes on the appropriate pharmacological management of insomnia.
(10)

A
How well did you know this?
1
Not at all
2
3
4
5
Perfectly
54
Q

a) There are various factors you need to take into consideration before prescribing any
psychotropic medication for a patient. Describe five of the most important and
relevant factors. (5)
b) Before concluding that a medication is having no therapeutic effect for a patient, you
need to ensure that the patient has been on an adequate trial of the medication in
question. Describe five of the most important and relevant conditions that must be
met in any medication trial for it to be considered adequate. (5)
c) Describe five of the most effective approaches to improve medication adherence. (5)
d) Write short notes on Lithium toxicity, under the following headings
i) Clinical presentation. (5)
ii) Management. (5)

A
How well did you know this?
1
Not at all
2
3
4
5
Perfectly
55
Q

You are called for a psychiatric consult on a middle-aged woman in the emergency
room after she had a generalised tonic clonic seizure in a Clicks pharmacy. She was
trying to obtain the non-benzodiazepine, Zolpidem (Stilnox) from the pharmacist
without a prescription in a very agitated state. She discloses to you that she has
been taking ten (10) Zolpidem tablets daily for three years and is dependent on. She
had failed to procure her drugs in the usual manner and had gone into withdrawal.
This was the first seizure she experienced
a) What are the most common symptoms of withdrawal from a CNS depressant? (10)
b) How would such a situation be managed in the short and long-term? (15)

A
How well did you know this?
1
Not at all
2
3
4
5
Perfectly
56
Q

Write short notes on the following
a) According to the MHCA (Mental Health Care Act of 2002) a patient can be admitted
to a hospital in four different ways. Describe each type of admission briefly. (10)
b) Discuss the necessary elements of informed consent. (10)
c) Explain what is meant by the term “competency or capacity for an informed decision”.
(5)

A
How well did you know this?
1
Not at all
2
3
4
5
Perfectly
57
Q

You are working as a medical officer at the psychiatry outpatient department and the
mother of a seven-year-old boy is referred with a problem of bedwetting
a) Describe the appropriate approach to this problem. (10)
b) Describe the non-pharmacological management strategies that you will discuss with
the mother. (8)

c) Which medication is most commonly used to treat Enuresis and what dose would
you recommend starting on? (2)
d) Classify elimination disorders. (5)

A
How well did you know this?
1
Not at all
2
3
4
5
Perfectly
58
Q

The feature that best differentiates bipolar disorder from MDD w mixed features is

A. Severity of manic symptoms
B. Number of manic symptoms
C. Duration of manic symptoms
D. Degree of functional impairment

A
How well did you know this?
1
Not at all
2
3
4
5
Perfectly
59
Q

Musa, 23yr male w schiz. Brother, father and uncle have treatment resistant schizophrenia on clozapine. He remains psychotic after 6 weeks on Haloperidol 10mg nocte. He’s adherent and not using substances. The next best option is

A. Increase haloperidol to 12.5mg nocte
B. Start him on Olanzapine 5mg nocte
C. Start him on Risp 2mg nocte
D. Start him on clozapine 12.5mg nocte

A
How well did you know this?
1
Not at all
2
3
4
5
Perfectly
60
Q

Accordong to MHCA 17 of 2002, HHE may limit intimate relationships of adults who

A. Are not voluntary MHCUs
B. Cannot consent
C. Are dangerous
D. Are admitted

A

Rights and Duties

R - respect for human dignity and privacy
E - exploitation and abuse
D - determination concerning mental health status will be based only on mental health and no other factor
D - disclosure of information - cannot disclose unless head of the national department, provincial department or HHE decides that failure to disclose will prejudice health of person or others.
L - limitation of intimate adult relationships - has a right to relationships when the MHCU is stable.
R - right to representation
K - knowledge of his/her rights
R - right to a discharge report

How well did you know this?
1
Not at all
2
3
4
5
Perfectly
61
Q

19yr old pulls her hair from eyebrows till there’s scarring and little eyebrow hair. Says her eyebrows are bushy, repulsive and make her look like a cave man. A picture before she started pulling seems normal to you. She most likely has

A. Trichotilomania
B. Body dysmorphic disorder
C. Delusional disorder
D. OCD

A
How well did you know this?
1
Not at all
2
3
4
5
Perfectly
62
Q

The feature which best differentiates childhood onset schizophrenia from autism spectrum disorder is

A. Degree of impairment
B. Family history
C. Hallucinations
D. Delusions
E. Negative symptoms

A
How well did you know this?
1
Not at all
2
3
4
5
Perfectly
63
Q

Tic disorders like Tourettes may be difficult to differentiate from a condition such as Obsessive Compulsive Disorder. The bestfeature that differentiates the two is:

A. Frequency of tics is higher in tic disorders
B. OCD patients have a need to perform compulsion “just right”
C. Tics never have a cognitive-based drive like obsessions
D. Compulsions reduce anxiety, tics do not

A
How well did you know this?
1
Not at all
2
3
4
5
Perfectly
64
Q

The class of medication with the most evidence for treating Autism Spectrum Disorders is
A. SSRIs
B. Second generation antipsychotics
C. Anti epileptic medications
D. None

A
How well did you know this?
1
Not at all
2
3
4
5
Perfectly
65
Q

The mainstay of treatment for tic disorders is psychotherapy which has similar effect size as medication. The most effective type of therapy is
A. Cognitive behavioural therapy
B. Habit reversaltherapy
C. Psychoeducation
D. Supportive psychotherapy

A
66
Q

The essential feature that differentiates cyclothymic disorder from bipolar disorder type 2 is

A. Longer duration of illness in cyclothymia
B. Multiple “episodes” in cyclothymia
C. Worse functional impairment in bipolar
D. Milder severity of mood symptoms in cyclothymia

A
67
Q

Tom had a panic attack at work and 1 at the gym. This is odd because his fears are around public transport and open space since he saw a taxi lose control in town. He actively avoids public transport and open spaces. This case meets dsm 5 criteria for

A. Agoraphobia
B. Panic disorder
C. PTSD
D. A combination of the above

A
68
Q

65yr male w SLE on tx w corticosteroids witnesses a major MVA. He begins to have disorganised speech for a few days then resolves. The most accurate diagnosis is

A. Schizophrenia
B. Psychotic disorder due to AMC - SLE
C. Steroid induced psychotic disorder
D. Brief psychotic disorder w marked stressor
E. Other specified schizophrenia spectrum and other psychotic disorders

A
69
Q

In which of the following ways do manic episodes differ from ADHD?

A. Manic episodes are more strongly associated w poor judgement
B. Manic episodes are more likely to involve excessive activity
C. Manic episodes have clearer symptomatic onsrt and offsets
D. Manic episodes are more likely to show a chronic course
E. Manic episodes first appear at an earlier age

A
70
Q

Musa, 23yr male w schiz. Brother, father and uncle have treatment resistant schizophrenia on clozapine. He remains psychotic after 6 weeks on Haloperidol 10mg nocte. He’s adherent and not using substances. The next best option is

A. Increase haloperidol to 12.5mg nocte
B. Start him on Olanzapine 5mg nocte
C. Start him on Risp 2mg nocte
D. Start him on clozapine 12.5mg nocte

A
71
Q

Mrs X lost intrest in previous activities she used to love, what’s the symptom?

A
72
Q

What do you see in CT Scan of a Schizophrenic patient

A
73
Q

35 yr old male patient who is diagnosed with schizophrenia at age 25 was treated for depression as he was withdrawn, not speaking to others (options were a. Negative symptoms of schizophrenia b. Prodome of schizophrenia c. MDD

A
74
Q

There is a patient who needs admission and you are the only doctor at district clinic, and 1 professional nurse and 1 staff nurse, who can fill the form 4 as the family can’t be found.

A
75
Q

Patient on venlaflaxine comes for follow up, What bedside investigations would you recommend.

A
76
Q

20 weeker pregnant lady 🤰 bipolar patient controlled for 5 years now concerned about taking pills which is the best mood stabilisers to use a. Carbamazepine b. Lamotrigine c. Sodium Valproate d. Lithium.

A
77
Q

A child who didn’t wanna separated to her mother had a problem even sleeping at friends place

A
78
Q

Diagnosis criteria of Encopresis how many times should the child experience it in a Month

A
79
Q

Distinctive facial features, including small eyes, an exceptionally thin upper lip, a short, upturned nose, and a smooth skin surface between the nose and upper lip which substance might have caused this features on a new born

A
80
Q

What is most common neupsychiatric manifestation of Stroke

A
81
Q

Criterion A for schizoaffective disorder requires an uninterrupted period of illness during which Criterion A for schizophrenia is met. Which of the following additional symptoms must be present to fulfil diagnostic criteria for schizoaffective disorder?
A. An anxiety episode—either panic or general anxiety.
B. Rapid eye movement (REM) sleep behaviour disorder.
C. A major depressive or manic episode.
D. Hypomania.
E. Cyclothymia.

A
82
Q

There is a requirement for a major depressive episode or a manic episode to be part of the symptom picture for a DSM-5 diagnosis of schizoaffective disorder. In order to separate schizoaffective disorder from depressive or bipolar disorder with psychotic features, which of the following symptoms must be present for at least 2 weeks in the absence of a major mood episode at some point during the lifetime duration of the illness?
A. Delusions or hallucinations.
B. Delusions or paranoia.
C. Regressed behaviour.
D. Projective identification.
E. Binge eating.

A
83
Q

55-year-old man with a known history of alcohol dependence and schizophrenia is brought to the emergency department because of frank delusions and visual hallucinations.
Which of the following would not be a diagnostic possibility for inclusion in the differential diagnosis?
A. Schizophrenia.
B. Substance/medication-induced psychotic disorder.
C. Alcohol dependence.
D. Psychotic disorder due to another medical condition.
E. Borderline personality disorder with psychotic features.

A
84
Q

Which of the following patient presentations would not be classified as psychotic for the purpose of diagnosing schizophrenia?
A. A patient is hearing a voice that tells him he is a special person.
B. A patient believes he is being followed by a secret police organization that is focused exclusively on him.
C. A patient has a flashback to a war experience that feels like it is happening again.
D. A patient cannot organize his thoughts and stops responding in the middle of an interview.
E. A patient presents wearing an automobile tire around his waist and gives no explanation.

A
85
Q

A 32-year-old man presents to the emergency department distressed and agitated. He reports that his sister has been killed in a car accident on a trip to South America. When asked how he found out, he says that he and his sister were very close and he “just knows it.” After putting him on the phone with his sister, who was comfortably staying with friends while on her trip, the man expressed relief that she was alive. Which of the following descriptions best fits this presentation?
A. He had a delusional belief, because he believed it was true without good warrant.
B. He did not have a delusional belief, because it changed in light of new evidence.
C. He had a grandiose delusion, because he believed he could know things happening far away.
D. He had a nihilistic delusion, because it involved an untrue, imagined catastrophe.
E. He did not have a delusion, because in some cultures people believe they can know things about family members outside of ordinary communications.

A
86
Q

64-year-old man who had been a widower for 3 months presents to the emergency department on the advice of his primary care physician after he reports
to the doctor that he hears his deceased wife’s voice calling his name when he looks through old photos, and sometimes as he is trying to fall asleep. His primary care physician tells him he is having a psychotic episode and needs to get a psychiatric evaluation. Which of the following statements correctly explains why these experiences are not considered to be psychotic?
A. The voice he hears is from a family member.
B. The experience occurs as he is falling asleep.
C. He can invoke her voice with certain activities.
D. The voice calls his name.
E. Both B and C.

A
87
Q

Which of the following does not represent a negative symptom of schizophrenia?
A. Affective flattening.
B. Decreased motivation.
C. Impoverished thought processes.
D. Sadness over loss of functionality.
E. Social disinterest.

A
88
Q

Schizophrenia spectrum and other psychotic disorders are defined by abnormalities in one or more of five domains, four of which are also considered psychotic symptoms. Which of the following is not considered a psychotic symptom?
A. Delusions.
B. Hallucinations.
C. Disorganized thinking.
D. Disorganized or abnormal motor behavior.
E. Avolition.

A
89
Q

What is the most common type of delusion?
A. Somatic delusion of distorted body appearance.
B. Grandiose delusion.
C. Thought insertion.
D. Persecutory delusion.
E. Former life regression.

A
90
Q

2.21 Label each of the following beliefs as a bizarre delusion, a nonbizarre delusion, or a nondelusion.
A. A 25-year-old law student believes he has uncovered the truth about JFK’s assassination and that CIA agents have been dispatched to follow him and monitor his Internet communications.
B. A 45-year-old homeless man presents to the psychiatric emergency room complaining of a skin rash. Upon removal of his clothes, it is seen that most of his body is wrapped in aluminum foil. The man explains that he is protecting himself from the electromagnetic ray guns that are constantly targeting him.
C. A 47-year-old unemployed plumber believes he has been elected to the House of Representatives. When the Capitol police evict him and bring him to the emergency department, he says that they are Tea Party activists who are merely impersonating police officers.
D. A 35-year-old high school physics teacher presents to your office with insomnia and tells you that he has discovered and memorized the formula for cold fusion energy, only to have the formula removed from his memory by telepathic aliens.
E. An 18-year-old recent immigrant from Eastern Europe believes that wearing certain colors will ward off the “evil eye” and prevent catastrophes that would otherwise occur.

A
91
Q

A 32-year-old man reports 1 week of feeling unusually irritable. During this time, he has increased energy and activity, sleeps less, and finds it difficult to sit still. He also is more talkative than usual and is easily distractible, to the point of finding it difficult to complete his work assignments. A physical examination and laboratory workup are negative for any medical cause of his symptoms and he takes no medications. What diagnosis best fits this clinical picture?
A. Manic episode.
B. Hypomanic episode.
C. Bipolar I disorder, with mixed features.
D. Major depressive episode.
E. Cyclothymic disorder.

A
92
Q

A 42-year-old man reports 1 week of increased activity associated with an elevated mood, a decreased need for sleep, and inflated self-esteem. Although the man does not object to his current state (“I’m getting a lot of work done!”), he is concerned because he recalls a similar episode 10 years ago during which he began to make imprudent business decisions. A physical examination and laboratory work are unrevealing for any medical cause of his symptoms. He had taken fluoxetine for a depressive episode but self-discontinued it 3 months ago because he felt that his mood was stable. Which diagnosis best fits this clinical picture?
A. Bipolar I disorder.
B. Bipolar II disorder.
C. Cyclothymic disorder.
D. Other specified bipolar disorder and related disorder.
E. Substance/medication-induced bipolar disorder.

A
93
Q

. A patient with a history of bipolar I disorder presents with a new-onset manic episode and is successfully treated with medication adjustment. He notes chronic depressive symptoms that, on reflection, long preceded his manic episodes. He describes these symptoms as “feeling down,” having decreased energy, and more often than not having no motivation. He denies other depressive symptoms but feels that these alone have been sufficient to negatively affect his marriage. Which diagnosis best fits this presentation?
A. Other specified bipolar and related disorder.
B. Bipolar I disorder, current or most recent episode depressed.
C. Cyclothymic disorder.
D. Bipolar I disorder and persistent depressive disorder (dysthymia).
E. Bipolar II disorder

A
94
Q

. In which of the following ways do manic episodes differ from attention-deficit/hyperactivity disorder (ADHD)?
A. Manic episodes are more strongly associated with poor judgment.
B. Manic episodes are more likely to involve excessive activity.
C. Manic episodes have clearer symptomatic onsets and offsets.
D. Manic episodes are more likely to show a chronic course.
E. Manic episodes first appear at an earlier age.

A
95
Q

A patient with a history of bipolar disorder reports experiencing 1 week of elevated and expansive mood. Evidence of which of the following would suggest that the patient is experiencing a hypomanic, rather than manic, episode?
A. Irritability.
B. Decreased need for sleep.
C. Increased productivity at work.
D. Psychotic symptoms.
E. Good insight into the illness

A
96
Q

A 25-year-old graduate student presents to a psychiatrist complaining of feeling down and “not enjoying anything.” Her symptoms began about a month ago, along with insomnia and poor appetite. She has little interest in activities and is having difficulty attending to her schoolwork. She recalls a similar episode 1 year ago that lasted about 2 months before improving without treatment. She also reports several episodes of increased energy in the past 2 years; these episodes usually last 1–2 weeks, during which time she is very productive, feels more social and outgoing, and tends to sleep less, although she feels energetic during the day. Friends tell her that she speaks more rapidly during these episodes but that they do not see it as off-putting and in fact think she seems more outgoing and clever. She has no medical problems and does not take any medications or abuse drugs or alcohol. What is the most likely diagnosis?
A. Bipolar I disorder, current episode depressed.
B. Bipolar II disorder, current episode depressed.
C. Bipolar I disorder, current episode unspecified.
D. Cyclothymic disorder.
E. Major depressive disorder

A
97
Q

How do the depressive episodes associated with bipolar II disorder differ from those associated with bipolar I disorder?
A. They are less frequent than those associated with bipolar I disorder.
B. They are lengthier than those associated with bipolar I disorder.
C. They are less disabling than those associated with bipolar I disorder.
D. They are less severe than those associated with bipolar I disorder.
E. They are rarely a reason for the patient to seek treatment.

A
98
Q

. How does the course of bipolar II disorder differ from the course of bipolar I disorder?
A. It is more chronic than the course of bipolar I disorder.
B. It is less episodic than the course of bipolar I disorder.
C. It involves longer asymptomatic periods than the course of bipolar I disorder.
D. It involves shorter symptomatic episodes than the course of bipolar I disorder.
E. It involves a much lower number of lifetime mood episodes than the course of bipolar I disorder.

A
99
Q

For an adolescent who presents with distractibility, which of the following additional features would suggest an association with bipolar II disorder rather than attention-deficit/hyperactivity disorder (ADHD)?
A. Rapid speech noted on examination.
B. A report of less need for sleep.
C. Complaints of racing thoughts.
D. Evidence that the symptoms are episodic.
E. Evidence that the symptoms represent the individual’s baseline behavior.

A
100
Q

A 50-year-old man with a history of a prior depressive episode is given an antidepressant by his family doctor to help with his depressive symptoms. Two weeks later, his doctor contacts you for a consultation because the patient now is euphoric, has increased energy, racing thoughts, psychomotor agitation, poor concentration and attention, pressured speech, and a decreased need to sleep. These symptoms began with the initiation of the patient’s new medication. The patient stopped the medication after 2 days, as he no longer felt depressed; however, the symptoms have continued ever since. What is the patient’s diagnosis?
A. Substance/medication-induced bipolar and related disorder.
B. Bipolar I disorder.
C. Bipolar II disorder.
D. Cyclothymic disorder.
E. Major depressive disorder.

A
101
Q

. In which of the following aspects does cyclothymic disorder differ from bipolar I disorder?
A. Duration.
B. Severity.
C. Age at onset.
D. Pervasiveness.
E. All of the above.

A
102
Q

Which of the following statements about how grief differs from a major depressive episode (MDE) is false?
A. In grief the predominant affect is feelings of emptiness and loss, while in MDE it is persistent depressed mood and the inability to anticipate happiness or pleasure.
B. The pain of grief may be accompanied by positive emotions and humor that are uncharacteristic of the pervasive unhappiness and misery characteristic of MDE.
C. The thought content associated with grief generally features a preoccupation with thoughts and memories of the deceased, rather than the self-critical or pessimistic ruminations seen in MDE.
D. In grief, feelings of worthlessness and self-loathing are common; in MDE, self-esteem is generally preserved.
E. If a bereaved individual thinks about death and dying, such thoughts are generally focused on the deceased and possibly about “joining” the deceased, whereas in MDE such thoughts are focused on ending one’s own life because of feeling worthless, undeserving of life, or unable to cope with the pain of depression.

A
103
Q

. A 50-year-old man presents with persistently depressed mood for several weeks that interferes with his ability to work. He has insomnia and fatigue, feels guilty, has thoughts he would be better off dead, and has thought about how he could die without anyone knowing it was a suicide. His wife informs you that he requests sex several times a day and that she thinks he may be going to “massage parlors” regularly, both of which are changes from his typical behavior. He has told her he has ideas for a “better Internet,” and he has invested thousands of dollars in software programs that he cannot use. She notes that he complains of fatigue but sleeps only 1 or 2 hours each night and seems to have tremendous energy during the day. Which diagnosis best fits this patient?
A. Manic episode.
B. Hypomanic episode.
C. Major depressive episode.
D. Major depressive episode, with mixed features.
E. Major depressive episode, with atypical features

A
104
Q

A depressed patient reports that he experiences no pleasure from his normally enjoyable activities. Which of the following additional symptoms would be required for this patient to qualify for a diagnosis of major depressive disorder with melancholic features?
A. Despondency, depression that is worse in the morning, and inability to fall asleep.
B. Depression that is worse in the evening, psychomotor agitation, and significant weight loss.
C. Inappropriate guilt, depression that is worse in the morning, and earlymorning awakening.
D. Significant weight gain, depression that is worse in the evening, and excessive guilt.
E. Despondency, significant weight gain, and psychomotor retardation

A
105
Q

. A 39-year-old woman reports that she became quite depressed in the winter last year when her company closed for the season, but she felt completely normal in the spring. She recalls experiencing several other episodes of depression over the past 5 years (for which she cannot identify a seasonal pattern) that would have met criteria for major depressive disorder. Which of the following correctly summarizes this patient’s eligibility for a diagnosis of “major depressive disorder, with seasonal pattern”?
A. She does not qualify for this diagnosis: the episode must start in the fall, and the patient must have no episodes that do not have a seasonal pattern.
B. She does qualify for this diagnosis: the single episode described started in the winter and ended in the spring.
C. She does not qualify for this diagnosis: the patient must have had two episodes with a seasonal relationship in the past 2 years and no nonseasonal episodes during that period.
D. She does qualify for this diagnosis: the symptoms described are related to psychosocial stressors.
E. She does qualify for this diagnosis: the symptoms are not related to bipolar I or bipolar II disorder.

A
106
Q

Which of the following statements about gender differences in suicide risk and suicide rates in major depressive disorder (MDD) is true?
A. The risk of suicide attempts and completions is higher for women.
B. The risk of suicide attempts and completions is higher for men.
C. The risk of suicide attempts and completions is equal for men and women.
D. The disparity in suicide rate by gender is much greater in individuals with MDD than in the general population.
E. The risk of suicide attempts is higher for women, but the risk of suicide completions is lower.

A
107
Q

A 12-year-old boy begins to have new episodes of temper outbursts that are out of proportion to the situation. Which of the following is not a diagnostic possibility for this patient?
A. Disruptive mood dysregulation disorder.
B. Bipolar disorder.
C. Oppositional defiant disorder.
D. Conduct disorder.
E. Attention-deficit/hyperactivity disorder.

A
108
Q

Which of the following features distinguishes disruptive mood dysregulation disorder (DMDD) from bipolar disorder in children?
A. Age at onset.
B. Gender of the child.
C. Irritability.
D. Chronicity.
E. Severity.

A
109
Q

. Children with disruptive mood dysregulation disorder are most likely to develop which of the following disorders in adulthood?
A. Bipolar I disorder.
B. Schizophrenia.
C. Bipolar II disorder.
D. Borderline personality disorder.
E. Unipolar depressive disorders

A
110
Q

An irritable 8-year-old child has a history of temper outbursts both at home and at school. What characteristic mood feature must be also present to qualify him for a diagnosis of disruptive mood dysregulation disorder?
A. The child’s mood between outbursts is typically euthymic.
B. The child’s mood between outbursts is typically hypomanic.
C. The child’s mood between outbursts is typically depressed.
D. The child’s mood between outbursts is typically irritable or angry.
E. The mood symptoms and temper outbursts must not have persisted for more than 6 months.

A
111
Q

Children with disruptive mood dysregulation disorder (DMDD) often meet criteria for what additional DSM-5 diagnosis?
A. Pediatric bipolar disorder.
B. Oppositional defiant disorder.
C. Schizophrenia.
D. Intermittent explosive disorder.
E. Major depressive disorder.

A
112
Q

The diagnostic criteria for disruptive mood dysregulation disorder (DMDD) state that the diagnosis should not be made for the first time before age 6 years or after 18 years (Criterion G). Which of the following statements best describes the rationale for this age range restriction?
A. Validity of the diagnosis has been established only in the age group 7–18 years.
B. The restriction represents an attempt to differentiate DMDD from bipolar disorder.
C. The restriction is based on existing genetic data.
D. The restriction represents an attempt to differentiate DMDD from intermittent explosive disorder.
E. The restriction represents an attempt to differentiate DMDD from autism spectrum disorder.

A
113
Q

A 9-year-old boy is brought in for evaluation because of explosive outbursts when he is frustrated with schoolwork. The parents report that their son is well behaved and pleasant at other times. Which diagnosis best fits this clinical picture?
A. Disruptive mood dysregulation disorder.
B. Pediatric bipolar disorder.
C. Intermittent explosive disorder.
D. Major depressive disorder.
E. Persistent depressive disorder (dysthymia).

A
114
Q

A 14-year-old boy describes himself as feeling “down” all of the time for the past year. He remembers feeling better while he was at camp for 4 weeks during the summer; however, the depressed mood returned when he came home. He reports poor concentration, feelings of hopelessness, and low self-esteem but denies suicidal ideation or changes in his appetite or sleep. What is the most likely diagnosis?
A. Major depressive disorder.
B. Disruptive mood dysregulation disorder.
C. Depressive episodes with short-duration hypomania.
D. Persistent depressive disorder (dysthymia), with early onset.
E. Schizoaffective disorder

A
115
Q

Which of the following is not required for a DSM-5 diagnosis of intellectual disability (intellectual developmental disorder)?
A. Full-scale IQ below 70.
B. Deficits in intellectual functions confirmed by clinical assessment.
C. Deficits in adaptive functioning that result in failure to meet developmental
and sociocultural standards for personal independence and social responsibility.
D. Symptom onset during the developmental period.

A
116
Q

A 7-year-old boy in second grade displays significant delays in his ability to reason, solve problems, and learn from his experiences. He has been slow to develop reading, writing, and mathematics skills in school. All through development, these skills lagged behind peers, although he is making slow progress. These deficits significantly impair his ability to play in an age-appropriate manner with peers and to begin to acquire independent skills at home. He requires on going assistance with basic skills (dressing, feeding, and bathing himself; doing any type of schoolwork) on a daily basis. Which of the following diagnoses best fits this presentation?
A. Childhood-onset major neurocognitive disorder.
B. Specific learning disorder.
C. Intellectual disability (intellectual developmental disorder).
Neurodevelopmental Disorders
D. Communication disorder.

A
117
Q

A 7-year-old boy in second grade displays significant delays in his ability to reason, solve problems, and learn from his experiences. He has been slow to develop reading, writing, and mathematics skills in school. All through development, these skills lagged behind peers, although he is making slow progress. These deficits significantly impair his ability to play in an age-appropriate manner with peers and to begin to acquire independent skills at home. He requires on going assistance with basic skills (dressing, feeding, and bathing himself; doing any type of schoolwork) on a daily basis. What is the appropriate severity rating for this patient’s current presentation?
A. Mild.
B. Moderate.
C. Severe.
D. Profound.

A
118
Q

. Which of the following is not a diagnostic feature of intellectual disability (intellectual developmental disorder)?
A. A full-scale IQ of less than 70.
B. Inability to perform complex daily living tasks (e.g., money management, medical decision making) without support.
C. Gullibility, with naiveté in social situations and a tendency to be easily led by others.
D. Lack of age-appropriate communication skills for social and interpersonal functioning.

A
119
Q

Which of the following statements about global developmental delay is true?
A. The diagnosis is typically made in children younger than 5 years of age.
B. The aetiology can usually be determined.
C. The prevalence is estimated to be between 0.5% and 2%.
D. The condition is progressive.

A
120
Q

Which of the following statements about the diagnosis of specific learning disorder is false?
A. Specific learning disorder is distinct from learning problems associated with a neurodegenerative cognitive disorder.
B. If intellectual disability (intellectual developmental disorder) is present, the learning difficulties must be in excess of those expected.
C. An uneven profile of abilities is typical in specific learning disorder.
D. There are four formal subtypes of specific learning disorder.

A
121
Q

Which of the following statements about comorbidity in specific learning disorder is true?
A. Attention-deficit/hyperactivity disorder (ADHD) does not co-occur with specific learning disorder more frequently than would be expected by chance.
B. Speech sound disorder and specific language impairments are not commonly comorbid with specific learning disorder.
C. Identified clusters of co-occurrences include severe reading disorders; fine motor problems and handwriting problems; and problems with arithmetic, reading, and gross motor planning.
D. The co-occurrence of specific learning disorder and specific language impairments has been shown in up to 20% of children with language problems.

A
122
Q

A 9-year-old boy cannot go to sleep without having a parent in his room. While falling asleep, he frequently awakens to check that a parent is still there. One parent usually stays until the boy falls asleep. If he wakes up alone during the night, he starts to panic and gets up to find his parents. He also reports frequent nightmares in which he or his parents are harmed. He occasionally calls out that he saw a strange figure peering into his dark room. The parents usually wake in the morning to find the boy asleep on the floor of their room. They once tried to leave him with a relative so they could go on a vacation; however, he became so distressed in anticipation of this that they cancelled their plans. What is the most likely diagnosis?
A. Specific phobia.
B. Nightmare disorder.
C. Delusional disorder.
D. Separation anxiety disorder.

A

D

123
Q

Which of the following statements best describes how panic attacks differ from panic disorder?
A. Panic attacks require fewer symptoms for a definitive diagnosis.
B. Panic attacks are discrete, occur suddenly, and are usually less severe.
C. Panic attacks are invariably unexpected.
D. Panic attacks represent a syndrome that can occur with a variety of other disorders.

A

D

124
Q

A 50-year-old man reports episodes in which he suddenly and unexpectedly awakens from sleep feeling a surge of intense fear that peaks within minutes.
During this time, he feels short of breath and has heart palpitations, sweating, and nausea. His medical history is significant only for hypertension, which is well controlled with hydrochlorothiazide. As a result of these symptoms, he has begun to have anticipatory anxiety associated with going to sleep. What is the most likely explanation for his symptoms?
A. Anxiety disorder due to another medical condition (hypertension).
B. Substance/medication-induced anxiety disorder.
C. Panic disorder.
D. Panic attacks.

A

D

125
Q

A 32-year-old woman reports sudden, unexpected episodes of intense anxiety, accompanied by headaches, a rapid pulse, nausea, and shortness of breath.
During the episodes she fears that she is dying, and she has presented several times to emergency departments. Each time she has been told that she is medically healthy; she is usually reassured for a time, but on the occurrence of a new episode she again becomes concerned that she has some severe medical problem. She was given lorazepam once but disliked the sedating effect and has not taken it again. She abstains from all medications and alcohol in an attempt to minimize potential causes for her attacks. What is the most likely explanation for her symptoms?
A. Panic disorder.
B. Somatic symptom disorder.
C. Anxiety due to another medical condition.
D. Illness anxiety disorder.

A

A

126
Q

A 65-year-old woman reports being housebound despite feeling physically healthy. Several years ago, she fell while shopping; although she sustained no injuries, the situation was so upsetting that she became extremely nervous when she had to leave her house unaccompanied. Because she has no children and few friends whom she can ask to accompany her, she is very distressed that she has few opportunities to venture outside her home. What is the most likely diagnosis?
A. Specific phobia, situational type.
B. Social anxiety disorder (social phobia).
C. Posttraumatic stress disorder.
D. Agoraphobia.

A

D

127
Q

Which of the following symptoms is a recognized consequence of the abrupt termination of daily or near-daily cannabis use?
A. Hallucinations.
B. Delusions.
C. Hunger.
D. Irritability.
E. Apathy

A

D

128
Q

Because opioid withdrawal and sedative, hypnotic, or anxiolytic withdrawal can involve very similar symptoms, distinguishing between the two can be difficult. Which of the following presenting symptoms would aid in making the correct diagnosis?
A. Nausea or vomiting.
B. Anxiety.
C. Yawning.
D. Restlessness or agitation.
E. Insomnia.

A

C

129
Q

A 27-year-old woman presents for psychiatric evaluation after almost hitting someone with her car while driving under the influence of marijuana. She reports that she was prompted to seek treatment by her husband, with whom she has had several conflicts over the past year about her ongoing marijuana use. She has continued to smoke two joints daily and drive while under the influence of marijuana since this event. What is the appropriate diagnosis?
A. Cannabis abuse.
B. Cannabis dependence.
C. Cannabis intoxication.
D. Cannabis use disorder.
E. Unspecified cannabis-related disorder.

A

D

130
Q

A 45-year-old man with a long-standing history of heavy alcohol use is referred for psychiatric evaluation after his recent admission to the hospital for acute hepatitis. The patient reports that he drank almost daily in college. Over the past 10 years, he has gradually increased his nightly alcohol intake from a single 6-pack to two 12-packs of beer, and this nightly drinking habit has resulted in his frequently oversleeping and missing work. He has tried to moderate his alcohol use on numerous occasions with little success, particularly after developing complications associated with alcoholic cirrhosis. The patient admits that he becomes anxious and gets hand tremors when he doesn’t drink. This patient meets the criteria for which of the following diagnoses?
A. Alcohol abuse.
B. Alcohol dependence.
C. Alcohol use disorder, mild.
D. Alcohol use disorder, moderate.
E. Alcohol use disorder, severe

A

E

131
Q

Which of the following statements about alcohol withdrawal is true?
A. Fewer than 10% of individuals undergoing alcohol withdrawal experience dramatic symptoms such as severe autonomic hyperactivity, tremors, or alcohol withdrawal delirium.
B. Delirium occurs in the majority of individuals who meet criteria for alcohol withdrawal.
C. Approximately 80% of all patients with alcohol use disorder will experience alcohol withdrawal.
D. Tonic-clonic seizures occur in about 15% of individuals who meet criteria for alcohol withdrawal.
E. Alcohol withdrawal symptoms typically begin between 24 and 48 hours after alcohol use has been stopped or reduced.

A

A

132
Q

How many remission specifiers are included in the DSM-5 diagnostic criteria for substance use disorders?
A. One.
B. Two.
C. Three.
D. Four.
E. Five.

A

B

133
Q

What is the most common co-occurring psychiatric diagnosis among individuals with a history of significant prenatal alcohol exposure?
A. Major depressive disorder.
B. Generalized anxiety disorder.
C. Attention-deficit/hyperactivity disorder.
D. Oppositional defiant disorder.
E. Substance use disorder.

A

C

134
Q
  1. A 7-year-old boy with mild to moderate developmental delay presents with a chronic history of wetting his clothes during the day about once weekly, even during school. He is now refusing to go to school for fear of wetting his pants and being ridiculed by his classmates. Which of the following statements accurately describes the diagnostic options regarding enuresis in this case?
    A. He should not be diagnosed with enuresis because the frequency is less than twice per week.
    B. He should be diagnosed with enuresis because the incontinence is resulting in impairment of age-appropriate role functioning.
    C. He should not be diagnosed with enuresis because his mental age is likely less than 5 years old.
    D. He should be diagnosed with enuresis, diurnal only subtype.
    E. He should not be diagnosed with enuresis because the events are restricted to the daytime.
A

C

135
Q
  1. Which of the following statements about enuresis is true?
    A. Over 60% of children diagnosed with enuresis have a comorbid DSM-5 disorder.
    B. Developmental delays are no more common in children with enuresis than in other children.
    C. Urinary tract infections are more common in children with enuresis.
    D. While embarrassing, enuresis has no effect on children’s self-esteem.
    E. Prevalence rates for enuresis at age 10 are similar to those at age 5.
A

C

136
Q

Which of the following statements about the diurnal-only subtype of enuresis is true?

A. This subtype is more common in males.
B. This subtype is more common after age 9 years.
C. This subtype is sometimes referred to as monosymptomatic enuresis.
D. This subtype is more common than the nocturnal-only subtype.
E. This subtype includes a subgroup of individuals with “voiding postponement,” in which micturition is consciously deferred because of a social reluctance to use the bathroom or to interrupt a play activity.

A

E

137
Q

Which of the following statements correctly identifies a distinction between primary enuresis and secondary enuresis?
A. Secondary enuresis is due to an identified medical condition; primary enuresis has no known etiology.
B. Children with secondary enuresis have higher rates of psychiatric comorbidity than do children with primary enuresis.
C. Primary enuresis has a typical onset at age 10, much later than the onset of secondary enuresis.
D. Primary enuresis is never preceded by a period of continence, whereas secondary enuresis is always preceded by a period of continence.
E. Unlike primary enuresis, secondary enuresis tends to persist into late adolescence.

A

D

138
Q

Which of the following statements correctly describes factors related to the etiology and/or onset of enuresis?
A. Enuresis has been shown to be heritable, with a child being twice as likely to have the diagnosis if either parent has had it.
B. Mode of toilet training or its neglect can affect rates of enuresis, as shown by high rates seen in orphanages.
C. In girls with enuresis, nocturnal enuresis is the more common form.
D. Rates of enuresis are much higher in European countries than in developing countries.
E. The development of modern diapers is believed to speed toilet training and reduce enuresis.

A

B

139
Q

A 6-year-old boy with mild to moderate developmental delay presents with a history of passing feces into his underwear during the day about once every 2 weeks, even during school. He is now refusing to go to school for fear of soiling his pants and being ridiculed by his classmates. Which of the following statements accurately describes the diagnostic options regarding encopresis in this case?
A. He should not be diagnosed with encopresis because the frequency is less than twice per week.
B. He should be diagnosed with encopresis because the incontinence is resulting in impairment of age-appropriate role functioning.
C. He should not be diagnosed with encopresis because his mental age is likely less than 4 years old.
D. He should be diagnosed with encopresis.
E. He should not be diagnosed with encopresis because the events are restricted to the daytime.

A

C

140
Q

Which of the following statements about encopresis is true?
A. When oppositional defiant disorder or conduct disorder is present, one cannot diagnose encopresis.
B. When constipation is present, one cannot diagnose encopresis.
C. Urinary tract infections can be comorbid with encopresis and are more common in girls.
D. Although it is embarrassing, encopresis has no effect on children’s self esteem.
E. Prevalence rates for encopresis at age 5 are estimated to be 5%.

A

C

141
Q

Which of the following statements correctly describes clinical aspects of the diagnosis of encopresis?
A. Encopresis with constipation and overflow incontinence is often involuntary.
B. Encopresis with constipation and overflow incontinence always involves well-formed stool.
C. Encopresis with constipation and overflow incontinence cannot be diagnosed if the behavior results from avoidance of defecation that develops for psychological reasons.
D. In encopresis with constipation and overflow incontinence, leakage usually occurs during sleep.
E. Encopresis with constipation and overflow incontinence rarely resolves after treatment of the constipation.

A

A

142
Q

The essential feature of the DSM-5 diagnosis of delirium is a disturbance in attention/awareness and in cognition that develops over a short period of time, represents a change from baseline, and tends to fluctuate in severity during the course of a day. Which of the following additional conditions must apply?
A. There must be laboratory evidence of an evolving dementia.
B. The disturbance must be associated with a disruption of the sleep-wake cycle.
C. The disturbance must not occur in the context of a severely reduced level of arousal, such as coma.
D. The disturbance must be a direct physiological consequence of a substance use disorder.
E. The disturbance must not be superimposed on a preexisting neurocognitive disorder.

A

C

143
Q

Both major and mild neurocognitive disorders can increase the risk of delirium and complicate its course. Traditionally, delirium is distinguished from dementia on the basis of the key features of acute onset, impairment in attention, and which of the following?
A. Fluctuating course.
B. Steady course.
C. Presence of mania.
D. Presence of depression.
E. Cog wheeling movements.

A

A

144
Q

A 79-year-old woman with a history of depression is being evaluated at a nursing home for a suspected urinary tract infection. She is easily distracted, perseverates on answers to questions, asks the same question repeatedly, is unable to focus, and cannot answer questions regarding orientation. The mental status changes evolved over a single day. Her family reports that they thought she “wasn’t herself” when they saw her the previous evening, but the nursing report this morning indicates that the patient was cordial and appropriate. What is the most likely diagnosis?
A. Major depressive disorder, recurrent episode.
B. Depressive disorder due to another medical condition.
C. Delirium.
D. Major depressive disorder, with anxious distress.
E. Obsessive-compulsive disorder.

A

C

145
Q

A 72-year-old man with no history of alcohol or other substance use disorders and no psychiatric history is brought to the emergency department (ED) because of transient episodes of unexplained loss of consciousness. His wife reports that he has experienced repeated falls and syncope over the past year, as well as auditory and visual hallucinations. A thorough workup for cardiac disease has found no evidence of structural heart disease or arrhythmias. In the ED, he is found to have severe autonomic dysfunction, including orthostatic hypotension and urinary incontinence. What is the best provisional diagnosis for this patient?
A. New-onset schizophrenia.
B. New-onset schizoaffective disorder.
C. Possible major or mild neurocognitive disorder with Lewy bodies.
D. Possible major or mild neurocognitive disorder due to Alzheimer’s disease.
E. New-onset seizure disorder.

A

C

146
Q

The diagnostic criteria for neurocognitive disorder (NCD) due to HIV infection include fulfillment of criteria for major or mild NCD and documented infection with human immunodeficiency virus (as confirmed by established laboratory methods). Which of the following is a prominent feature of NCD due to HIV infection?
A. Impairment in executive functioning.
B. Conspicuous aphasia.
C. Significant delusions and hallucinations at onset of the disorder.
D. Marked difficulty with recall of learned information.
E. Rapid progression to profound neurocognitive impairment.

A

A

147
Q

In addition to documented infection with HIV and fulfillment of criteria for major or mild neurocognitive disorder (NCD), what other requirement must be met to qualify for a diagnosis of major or mild NCD due to HIV infection?
A. Presence of HIV in the cerebrospinal fluid.
B. A pattern of cognitive impairment characterized by early predominance of aphasia and impaired memory for previously learned information.
C. Presence of progressive multifocal leukoencephalopathy.
D. Inability to attribute the NCD to non-HIV conditions (including secondary brain diseases), another medical condition, or a mental disorder.
E. Presence of Kayser-Fleisher rings.

A

D

148
Q

Which of the following features characterizes alcohol-induced major or mild neurocognitive disorder, amnestic-confabulatory type?
A. Amnesia for new information and confabulation.
B. Seizures.
C. Amnesia for previously learned information and downward gaze paralysis.
D. Aphasia.
E. Anosognosia and apraxia.

A

A

149
Q

Major and mild neurocognitive disorders (NCDs) exist on a spectrum of cognitive and functional impairment. Which of the following constitutes an important threshold differentiating the two diagnoses?
A. Whether or not the individual is concerned about the decline in cognitive function.
B. Whether or not there is impairment in cognitive performance as measured by standardized testing or clinical assessment.
C. Whether or not the cognitive impairment is sufficient to interfere with independent completion of activities of daily living.
D. Whether or not the cognitive deficits occur exclusively in the context of a delirium.
E. Whether or not the cognitive deficits are better explained by another mental disorder.

A

C

150
Q

A 25-year-old woman, new to your practice, tells you that a little more than 3 months ago she was accosted on her way home. The attacker told her he had a gun, was going to rape her, and would shoot her if she resisted. He walked her toward an alley. She was sure he would kill her afterward no matter what she did, and therefore she pushed away from him, aware that she might be shot. She was able to escape unharmed. She describes not being able to fall asleep for the first 2 nights after the attack and of avoiding that particular street in her neighborhood for 2 days following the event. She thinks that the attacker might have touched her breasts but cannot remember for sure. She has recently started feeling anxious all of the time and is tearful, and she has stopped going to work. She fears that something about her makes her “look like a victim.” What is the most likely diagnosis?
A. Posttraumatic stress disorder.
B. Acute stress disorder.
C. Adjustment disorder.
E. Personality disorder

A

C

151
Q

After a routine chest X ray, a 53-year-old man with a history of heavy cigarette use is told that he has a suspicious lesion in his lung. A bronchoscopy confirms the diagnosis of adenocarcinoma. The man delays scheduling a follow-up appointment with the oncologist for more than 2 weeks, describes feeling as if “all of this is not real,” is having nightly dreams of seeing his own tombstone, and is experiencing intrusive flashbacks to the moment when he heard the physician saying, “The tests strongly suggest that you have cancer of the lung.” He is tearful and is convinced he will die. He also feels intense guilt that his smoking caused the cancer and expresses the thought that he “deserves” to have cancer. What diagnosis best fits this clinical picture?
A. Acute stress disorder.
B. Posttraumatic stress disorder.
C. Adjustment disorder.
D. Major depressive disorder.
E. Generalized anxiety disorder

A

C

152
Q

Criterion B for acute stress disorder requires the presence of nine (or more) symptoms from any of five categories of response. Which of the following is not one of these five categories?
A. Intrusion.
B. Dissociation.
C. Confusion.
D. Avoidance.
E. Arousal.

A

C

153
Q

Which of the following stressful situations would meet Criterion A for the diagnosis of acute stress disorder (ASD)?
A. Finding out that one’s spouse has been fired.
B. Failing an important final examination.
C. Receiving a serious medical diagnosis.
D. Being in the cross fire of a police shootout but not being harmed.
E. Being in a subway train that gets stuck between stations.

A

D

154
Q

Following discharge from the hospital, a 22-year-old man describes vivid and intrusive memories of his stay in the intensive care unit (ICU), where he received treatment for smoke inhalation. Now at home, he states that he has memories of people being tortured and hearing their screams. He dreams of this every night, waking from sleep in a terror. He talks about not feeling like himself after the experience, finding little pleasure in life after what happened to him, and being easily angered by his family; in addition, he avoids his physician out of fear that he will be told he needs to return to the ICU. What is the most likely explanation for this patient’s symptoms?
A. He has acute stress disorder because his life was in danger during the ICU stay.
B. He has posttraumatic stress disorder because his life was in danger during the ICU stay.
C. He has a delirium persisting from the ICU stay.
D. He had a delirium in the ICU and now has an adjustment disorder.
E. He has a psychotic disorder.

A

D

155
Q

Which of the following experiences would not qualify as exposure to a traumatic event (Criterion A) in the diagnosis of acute stress disorder or posttraumatic stress disorder?
A. Hearing that one’s brother was killed in combat.
B. Hearing that one’s close childhood friend survived a motor vehicle accident but is paralyzed.
C. Hearing that one’s child has been kidnapped.
D. Hearing that one’s company had suddenly closed.
E. Hearing that one’s spouse has been shot.

A

D

156
Q

A 31-year-old man narrowly escapes (without injury) from a house fire caused when he dropped the lighter while trying to light his crack pipe. Six weeks later, while smoking crack, he thinks he smells smoke and runs from the building in a panic, shouting, “It’s on fire!” Which of the following symptoms or circumstances would rule out a diagnosis of posttraumatic stress disorder (PTSD) for this patient?
A. Having difficulty falling asleep.
B. Being uninterested in going back to work.
C. Inappropriately getting angry at family members.
D. Experiencing symptoms only when smoking crack cocaine.
E. Concluding that “the world is completely dangerous.”

A

D

157
Q

Criterion A4 of posttraumatic stress disorder requires “Experiencing repeated or extreme exposure to aversive details of the traumatic event.” Which of the following would not qualify as an experiencing trauma under this criterion?
A. A police officer reviewing surveillance videotapes of homicides to identify perpetrators.
B. A social worker interviewing children who have been sexually abused and obtaining the details of the abuse.
C. A soldier sifting through the rubble of a collapsed building to retrieve remains of comrades.
D. A college student at a film festival watching a series of violent movies that contain graphic rape scenes.

A

D

158
Q

A psychologist working with victims of torture who are seeking political asylum in the United States.
Which of the following statements about gender differences in the risk of developing posttraumatic stress disorder (PTSD) is true?
A. The risk is lower in females in preschool-age populations.
B. The risk is higher in females across the life span.
C. The risk is higher in males in elderly populations.
D. The risk is lower in middle-aged females than in middle-aged males.
E. The risk is higher in males across the life span.

A

B

159
Q

5-year-old child was present when her babysitter was sexually assaulted. Which of the following symptoms would be most suggestive of posttraumatic stress disorder (PTSD) in this child?
A. Playing normally with toys.
B. Having dreams about princesses and castles.
C. Taking the clothing off her dolls while playing.
D. Expressing no fear when talking about the event.
E. Talking about the event with her parents.

A

C

160
Q

Which of the following statements about risk factors for developing posttraumatic stress disorder (PTSD) is true?
A. Sustaining personal injury does not affect the risk of developing PTSD.
B. Severity of the trauma influences the risk of developing PTSD.
C. Dissociation has no impact on the risk of developing PTSD.
D. Perceived life threat is the only risk factor for developing PTSD.
E. Prior mental disorders have little influence on the risk of developing PTSD.

A

B